In: Finance
Consider the three stocks in the following table.
Ptrepresents price at time t, and
Qtrepresents shares outstanding at time
t. There are three time points 0, 1, and 2.
P0 | Q0 | P1 | Q1 | P2 | Q2 | |
A | 90 | 100 | 95 | 100 | 100 | 100 |
B | 50 | 200 | 45 | 200 | 45 | 200 |
C | 100 | 200 | 110 | 200 | 105 | 200 |
a) Form an equally weighted portfolio in stocks A, B, C at time 0
and hold it to time 1.
Calculate the portfolio return from 0 to 1.
Keep two decimal places.
b) Form an equally weighted portfolio in stocks B, C (No Stock A) at time 1 and hold it to time 2.
Calculate the portfolio return from 1 to 2.
Keep two decimal places.
Solution (a)
Stock | Weights | P0 | Q0 | Purchase Cost | P1 | Q1 | Sale Value | Return | Return * Weight |
A | 0.33 | 90.00 | 100.00 | 9,000.00 | 95.00 | 100.00 | 9,500.00 | 500.00 | 166.67 |
B | 0.33 | 50.00 | 200.00 | 10,000.00 | 45.00 | 200.00 | 9,000.00 | -1,000.00 | -333.33 |
C | 0.33 | 100.00 | 200.00 | 20,000.00 | 110.00 | 200.00 | 22,000.00 | 2,000.00 | 666.67 |
Portfolio Return | 500.00 |
Return in %
Stock A = 166.67 / 9,000 * 100 = 1.85%
Stock B = -333.33 / 10,000 * 100 = -3.33%
Stock C = 666.67 / 20,000 * 100 = -3.33%
Total Portfolio Return = 1.85% (1.85% + (-3.33%) + 3.33%)
Solution (b)
Stock | Weights | P1 | Q1 | Purchase Cost | P2 | Q2 | Sale Value | Return | Return * Weight |
B | 0.50 | 45.00 | 200.00 | 9,000.00 | 45.00 | 200.00 | 9,000.00 | - | - |
C | 0.50 | 110.00 | 200.00 | 22,000.00 | 105.00 | 200.00 | 21,000.00 | -1,000.00 | -500.00 |
Portfolio Return | -500.00 |
Return in %
Stock B = 0 / 9,000 * 100 = 0%
Stock C = -500 / 22,000 * 100 = -2.27%
Total Portfolio Return = -2.27% (0% + (-2.27%))